Bold Face CR

This topic has expert replies
Master | Next Rank: 500 Posts
Posts: 115
Joined: Thu Mar 24, 2011 3:49 am
Thanked: 1 times

Bold Face CR

by vidhya16 » Sun Apr 17, 2011 8:02 am
3. The interstitial nucleus, a sub-region of the brain¡¯s hypothalamus, is typically smaller for male cats than for female cats. A neurobiologist performed autopsies on male cats who died from disease X, a disease affecting no more than 0.5 percent of male cats, and found that these male cats had interstitial nuclei that were as large as those generally found in female cats. Thus, the size of the interstitial nucleus determines whether or not male cats can contract disease X, but, the hypothalamus is known not to be causally linked to disease Y, and disease X is a subtype of disease Y.

The two portions in boldface play which of the following roles?

(A) The first is a fact in support of the consideration that is one of two points of this argument; the second is the alternative point that weighs against the first.

(B) The first is an evidence that supports the consideration that the argument includes; the second is the fact that weighs against that consideration that could be drawn from the first.

(C) The first is a general principle that is against the conclusion; the second is that conclusion.

(D) The first is an evidence that supports the conclusion; the second is an exceptional example.

(E) The first is a fact in support of the conclusion that the argument depends on; the second is a fact that is against the first one.

IMO (E).

Legendary Member
Posts: 1574
Joined: Fri Jan 28, 2011 2:52 am
Thanked: 88 times
Followed by:13 members

by aspirant2011 » Sun Apr 17, 2011 9:15 am
i feel its between B and D and I am more inclined towards D than B........because "these male cats had interstitial nuclei that were as large as those generally found in female cats" ------> part is an evidence which is supporting the conclusion "Thus, the size of the interstitial nucleus determines whether or not male cats can contract disease X".........

whats the OA?????

Master | Next Rank: 500 Posts
Posts: 115
Joined: Thu Mar 24, 2011 3:49 am
Thanked: 1 times

by vidhya16 » Sun Apr 17, 2011 10:55 am
I will wait for a day and post the OA.

Ta

Master | Next Rank: 500 Posts
Posts: 184
Joined: Sat Apr 14, 2007 9:23 am
Location: Madison, WI
Thanked: 17 times

by ldoolitt » Sun Apr 17, 2011 1:44 pm
Me thinks this is a poor question. Is the source of this LSAT or otherwise? The wording strikes me as not being GMAT typical (for example, B says "supports the consideration that the argument includes", what consideration is it referring to? its ambiguous)

I would say you could safely ignore this question.

But for what it is worth, I weakly conclude that (A) is correct.

User avatar
Master | Next Rank: 500 Posts
Posts: 234
Joined: Tue Feb 22, 2011 5:02 am
Thanked: 5 times
Followed by:3 members

by champmag » Mon Apr 18, 2011 10:38 pm
IMO:D

User avatar
Legendary Member
Posts: 979
Joined: Tue Apr 14, 2009 1:38 am
Location: Hyderabad, India
Thanked: 49 times
Followed by:12 members
GMAT Score:700

by bubbliiiiiiii » Tue Apr 19, 2011 1:17 am
Confused between D/E.
Regards,

Pranay

User avatar
Legendary Member
Posts: 1101
Joined: Fri Jan 28, 2011 7:26 am
Thanked: 47 times
Followed by:13 members
GMAT Score:640

by HSPA » Tue Apr 19, 2011 2:13 am
B(M) < B(F);
if(x)
b(M) == b(F); // A premise

Y = X+k; //fact
b!=Y; // Another premise which hits the stem
-------------------------------------------
A/C/E are out because first is a premise/evidence.
B > D

IMO B
First take: 640 (50M, 27V) - RC needs 300% improvement
Second take: coming soon..
Regards,
HSPA.

User avatar
Legendary Member
Posts: 979
Joined: Tue Apr 14, 2009 1:38 am
Location: Hyderabad, India
Thanked: 49 times
Followed by:12 members
GMAT Score:700

by bubbliiiiiiii » Tue Apr 19, 2011 2:18 am
HSPA wrote:B(M) < B(F);
if(x)
b(M) == b(F); // A premise

Y = X+k; //fact
b!=Y; // Another premise which hits the stem
-------------------------------------------
A/C/E are out because first is a premise/evidence.
B > D

IMO B
How much time did it take to solve?
Regards,

Pranay

User avatar
Legendary Member
Posts: 1101
Joined: Fri Jan 28, 2011 7:26 am
Thanked: 47 times
Followed by:13 members
GMAT Score:640

by HSPA » Tue Apr 19, 2011 2:20 am
I took 1.56 min to come to a conclusion
First take: 640 (50M, 27V) - RC needs 300% improvement
Second take: coming soon..
Regards,
HSPA.

Legendary Member
Posts: 1574
Joined: Fri Jan 28, 2011 2:52 am
Thanked: 88 times
Followed by:13 members

by aspirant2011 » Tue Apr 19, 2011 7:50 am
yup i agree with HSPA, the answer should be B..............

GMAT Instructor
Posts: 1302
Joined: Mon Oct 19, 2009 2:13 pm
Location: Toronto
Thanked: 539 times
Followed by:164 members
GMAT Score:800

by Testluv » Tue Apr 19, 2011 7:59 am
To the OP: where did you get this question? I recognize the stimulus as an LSAT question but LSAT doesn't use boldface.

I think the stimulus is from LSAT but the rest of the question is "home-made." If it is homemade (or from 1000CR), then it is not worth anyone's attention.
Kaplan Teacher in Toronto

Master | Next Rank: 500 Posts
Posts: 115
Joined: Thu Mar 24, 2011 3:49 am
Thanked: 1 times

by vidhya16 » Tue Apr 19, 2011 9:35 am
TestLuv,

Question is from Urch.com. You can search Boldface CR! and find a set of 10 question.

Ta

GMAT Instructor
Posts: 1302
Joined: Mon Oct 19, 2009 2:13 pm
Location: Toronto
Thanked: 539 times
Followed by:164 members
GMAT Score:800

by Testluv » Tue Apr 19, 2011 12:50 pm
vidhya16 wrote:TestLuv,

Question is from Urch.com. You can search Boldface CR! and find a set of 10 question.

Ta
Then, I wouldn't practice on questions from this source.
Kaplan Teacher in Toronto

User avatar
GMAT Instructor
Posts: 2193
Joined: Mon Feb 22, 2010 6:30 pm
Location: Vermont and Boston, MA
Thanked: 1186 times
Followed by:512 members
GMAT Score:770

by David@VeritasPrep » Tue Apr 19, 2011 1:41 pm
TestLuv is right on this, but too kind!

This is an LSAT question from June of 1999, the 2nd Logical Reasoning section question #25.

AT LEAST THE STIMULUS IS!!!

Some rotten thief has taken an LSAT question stimulus - this was originally a weaken question - and has made up his own answers. None of these answers are correct - what is more none of these answers are even logical. What is this ridiculous answer choice B "(B) The first is an evidence that supports the consideration that the argument includes; the second is the fact that weighs against that consideration that could be drawn from the first." This is some junk made up by someone who wants to sound complicated. BAH!

If I sound upset it is because I am. The logic of GMAT critical reasoning questions (and LSAT questions as well) is a beautiful thing. And I just hate it when these questions come around that confuse people and cast a bad light on my favorite GMAT topic - critical reasoning.

Reputable sources people please. Reputable sources. Which include -- Official questions. Questions from the major companies - all of which are represented by experts on this forum; experts who will ANSWER FOR THE QUALITY OF THEIR COMPANIES' QUESTIONS. LSAT questions are good as well as long as they are not stolen.

You will say, but I have practiced every question from the Official Guides, every question from Kaplan, Manhattan, Veritas, etc. Well then it is time to go and take the GMAT! You have studied enough! And if you still need to practice go back through the materials that you have. More material is not better - especially not when it will confuse you terribly.

So yes, as TestLuv said, "I wouldn't practice questions from this source."
Veritas Prep | GMAT Instructor

Veritas Prep Reviews
Save $100 off any live Veritas Prep GMAT Course

Master | Next Rank: 500 Posts
Posts: 115
Joined: Thu Mar 24, 2011 3:49 am
Thanked: 1 times

by vidhya16 » Wed Apr 20, 2011 5:04 am
David,

Thank you for the detailed reply. Im very new to GMAT so couldnt figure this out. I will take your point and practice only from trusted sources.

Ta